Questions tagged [lower-bounds]

The tag has no usage guidance.

51 questions with no upvoted or accepted answers
Filter by
Sorted by
Tagged with
12 votes
0 answers
801 views

Lower bounds for linear forms of logarithms (a la Baker)?

Let $\lambda_1$, $\lambda_2$, and $a$ be three fixed complex algebraic numbers. For a given integer $n$, write $\Theta(n) = \arg(a \lambda_1^n + \lambda_2^n)$. Assuming $\Theta(n)$ is not zero, I am ...
Joel Ouaknine's user avatar
11 votes
0 answers
524 views

Bounding the probability that a random variable is maximal

Question: Suppose we have $N$ independent random variables $X_1$, $\ldots$, $X_N$ with finite means $\mu_1 \leq \ldots \leq \mu_N$ and variances $\sigma_1^2$, $\ldots$, $\sigma_N^2$. I am looking ...
MLS's user avatar
  • 119
8 votes
0 answers
265 views

Restricted divisor summatory function

I have a problem that boils down to prove that the succession $\{a_n\}$ tends to infinity, where $$a_n:=1+\sum_{0\leq j<n}D_{2j+1}(n-j)$$ and $D_{m}(n)$ is the number of divisors $d>1$ of $n$ ...
Nick Belane's user avatar
5 votes
0 answers
84 views

Smaller root of a difference of products of polynomials with integer bounded coefficients

Is there a positive constant $K>0$ such that for every polynomials $f_1,\dots,f_4 \in\mathbb{Z}[X]$ with coefficients in {-1,0,1}, every positive root $x$ of the polynomial $$g=f_1f_2-f_3f_4$$ ...
A. Lampadophore's user avatar
4 votes
0 answers
123 views

Log of a truncated binomial

Let $X$ follow a binomial distribution with $n$ trials and success probability $p$, and let $0\leq k\leq n$. Are there any natural approximations or bounds for the ratio $$\frac{\boldsymbol{E}\log\...
Tom Solberg's user avatar
  • 3,929
3 votes
0 answers
64 views

Does this information theoretical thought experiment have a name or corresponding area of research?

I came up with the following thought experiment in my research in order to better understand the way Turing machines can transfer information through their tapes (the motivation is detailed below, isn'...
exfret's user avatar
  • 479
3 votes
0 answers
90 views

Probability of winning a $k$-rounds coin toss game

Let $p,q \in [0,1]$ with $p>q$. I denote by $B_k(p), B_k(q)$ two independent random variables following the binomial distribution, with parameters $(k,p)$ and $(k,q)$ respectively. Informal ...
Argemione's user avatar
  • 131
3 votes
0 answers
666 views

Counting / characterizing the isolated points of a particular algebraic variety

I'm not a professional geometer / topologist, so please thanks for your patience :) Setup The following questions are the first in a series of steps I'm undertaking in an attempt to break down a ...
dohmatob's user avatar
  • 6,726
3 votes
0 answers
209 views

Lower bound on number of $r$-regular graphs witn $n$ vertices

Consider the set of $r$-regular labeled graphs with $n$ vertices. There are results on its asymptotic size (see for instance this question on MO). Is there a known, explicit lower bound on that size, ...
Joe Dohn's user avatar
3 votes
0 answers
258 views

A generalization of coupon collector problem - $\geq1$ pick per experiment

Mix $T\geq1$ coupons numbered $1$ to $T$ with a set of $S\geq0$ number of dummy coupons with no numbers. Select $N\geq1$ coupons at each trial at random and put them back. $N=1$ is standard coupon ...
user avatar
3 votes
0 answers
289 views

Geometric/Analytic techniques for constructive and asymptotic bounds in the Lee metric

Slight extension of cross posting from https://cstheory.stackexchange.com/questions/7408/lee-metric-gilbert-varshamov-and-hamming-bounds-for-larger-relative-distance-rang (closed there) The ...
user16007's user avatar
  • 800
2 votes
0 answers
190 views

When does Le Cam's method give tight lower bounds for distribution testing?

In the context of statistical estimation or distribution testing, Le Cam's method is a way to prove lower bounds on the minimax sample complexity ([1,2,3,4], further details below). My question is: ...
π314's user avatar
  • 33
2 votes
0 answers
141 views

How is the Cauchy-Schwarz equality and the assumption on the support of $g$ used to derive this bound?

I am currently reading On Uniqueness Properties of Solutions of Schrödinger Equations and a having trouble understanding a claim made on page 1819. Context from the paper: let $g\in C^\infty_0(\...
Dispersion's user avatar
2 votes
0 answers
286 views

Extension of the Gershgorin circle theorem for symmetric matrices and localization of positive eigenvalues

In mathematics, the Gershgorin circle theorem can be used to localize eigenvalues of a matrix (including symmetric). Let $A$ be a real symmetry $n × n$ matrix, with entries $a_{ij}$. For $i∈{1,…,n}$ ...
dtn's user avatar
  • 145
2 votes
0 answers
204 views

Gershgorin-type bounds for smallest eigenvalue of positive-definite matrix

I would like to know if there are known results for bounding eigenvalues of positive-definite matrices, in particular gram matrices $AA^\top$ based on easily computable functions of $A$. Gershgorin ...
trenta3's user avatar
  • 109
2 votes
0 answers
150 views

Size of an “average” ϵ-net on the unit sphere

This is a question I originally asked on math.stackexchange, but didn't receive a satisfying answer. Let $\epsilon>0$ and consider constructing a set $S_\epsilon\subseteq S^{d-1}$ of points on the ...
R B's user avatar
  • 608
2 votes
0 answers
329 views

Mertens Bound and the Riemann Hypothesis

Let $M(x)$ denote the Mertens function ($M(x)=\sum_{i=1}^{x}\mu(i)$ where $\mu(i)$ is the Möbius function) and let $\Lambda(i)$ denote the Mangoldt function ($\Lambda(i)$ equals $\log(p)$ if $i=p^{m}$ ...
Sourangshu Ghosh's user avatar
2 votes
0 answers
591 views

An interesting sequence of numbers arising from the Riemann hypothesis

A very good coincidence occurred today with me. While just plotting random functions in Mathematica, I entered this command: ...
user avatar
2 votes
0 answers
390 views

Bound on eigenvalues of A+B (Hermitan matrices) which is better that the Lidskii and Weyl bounds

I have two positive definite $N\times N$ Hermitian matrices $A$ and $A$ and am interested in bounding the eigenvalues of $A+B$ in terms of the eigenvalues of $A$ and $B$. Let $\lambda_k(\cdot)$ be ...
Tom Steiglitz's user avatar
2 votes
0 answers
65 views

Minimal condition $\sum \rho_{ij} \Psi_{ij} s_i s_j < \sum s_i s_j $, $\mid \rho_{ij} \mid \leq 1$, $s_i \in \mathbb R$ and $\Psi_{ij} \in \{0,1\}$

Consider a sequence of real number $\{s_i\}_{i\leq n}$. Now consider the real numbers $F$, $G$ and $\alpha$ defined below $$F= \sqrt{ \left( \sum ~\rho_{ij} ~\Psi_{ij}~ s_i ~s_j \right)^+}, $$ $$G = ...
user50278's user avatar
  • 123
2 votes
0 answers
133 views

Lower asymptotic bounds for the derivative of Laguerre polynomials

Let $ L_{d}^{(1)}(x)$ denote the generalized Laguerre polynomial of degree $d$ and order $\alpha=1$. Clearly, since all the roots $r_1,\dots,r_d$ of $L_{d}^{(1)}$ are simple, there exists a strictly ...
dima's user avatar
  • 949
1 vote
0 answers
84 views

Obstacles to computing $\pi(n)$ in $O(n^{2/3-\epsilon})$ time

Edit: Apologies, as mentioned in the comments I failed to notice the analytic algorithms that take $O(n^{1/2+\epsilon})$ time, so this question doesn’t make much sense. It’s possible there is a ...
Geoffrey Irving's user avatar
1 vote
0 answers
106 views

What is known about the average growth rate of the denominators of $n$ Egyptian fractions summing to one?

Motivation In the following question posted here on MO and over at MSE, user Noah Schweber asks about a weighted count on Egyptian fraction representations (EFRs). To that end, he defines the ...
Max Muller's user avatar
  • 4,575
1 vote
0 answers
303 views

Lower bound on the sum of the product of random variables

Let $X_i$ be the $i$-th element of the vector $X=(X_1, ..., X_m)$ of i.i.d. random variables. I am looking for a lower bound for the expression $\mathbb{P}((\sum^n_{i=1}\prod^{m_i}_{j=1}(X_j))^2 \geq ...
Scriddie's user avatar
  • 129
1 vote
0 answers
197 views

"Tails" of a multinomial distribution

Let $X_1,\dots,X_N$ denote a collection of independent samples of a uniform multinomial random variable in $\mathbb{Z}^k$, with the number of trials equal to $n\ll k$. (By "uniform", I mean ...
Tom Solberg's user avatar
  • 3,929
1 vote
0 answers
32 views

Are there known bounds on these ratios of chromatic polynomials?

The chromatic polynomial $P(G, \lambda)$ gives the number of proper vertex colorings of the graph $G$ with $\lambda$ colors. I'm interested in how many possible colorings you loose when you add an ...
M. Stern's user avatar
  • 111
1 vote
0 answers
45 views

Analytic lower-bound for minimal value of $\|x\|^2$ such that $\|Cx-b\|^2 \le c^2$ (a hyperellipsoid)

Let $C$ be an $n \times p$ matrix and $b$ be a column vector of length $n$, and $c>0$. Let $E := \{x \in \mathbb R^p \mid \|Cx-b\| \le c\}$, a hyperellipsoid in nonstandard position. Question 1. ...
dohmatob's user avatar
  • 6,726
1 vote
0 answers
31 views

Given a unit vector $x\in\mathbb R^d$, what is the worst possible within-cluster sum of squares for 2-means clustering?

This is a question I originally posted to math.stackexchange.com but it didn't attract any answers, and I was wondering if someone here can help. Consider a unit vector $x\in\mathbb R^d$ ($\|x\|_2=1$)...
M A's user avatar
  • 127
1 vote
0 answers
121 views

A lower-bound on matrix-function with vector product

I am currently trying to show that a sequence of homeomorphisms converges to some limiting homeomorphism using Anderson's the inductive convergence criterion. However I can't explicitly compute the ...
ABIM's user avatar
  • 4,969
1 vote
0 answers
110 views

Bounds on spectral radius using chromatic number

I am struggling with this question: If I have a connected graph $G$ on $n$ vertices and $m$ edges with chromatic number $d$ then how can I give a bound(lower and upper) on its spectral radius in ...
Learnmore's user avatar
  • 135
1 vote
0 answers
105 views

Bounding quantiles of the noncentral chi distribution

I need to bound the empirical quantiles for a noncentral chi distribution (not chi-squared) $\chi_\nu(\lambda)$, where $\nu$ is the number of degrees of freedom and $\lambda$ the non-centrality ...
etal's user avatar
  • 162
1 vote
0 answers
61 views

Cramer Rao bound for relative estimation

I have an observed vector ${\bf y}$ from which I would like to estimate a parameter vector ${\bf c}$ (denote the estimate $\hat{{\bf c}}$). A feature of our estimation problem is that the involved ...
Sancho Panza's user avatar
1 vote
0 answers
117 views

Finding the infimum of the range of a certain non-negative function associated to a $ C^{*} $-algebra

Let $ A $ be a non-trivial $ C^{*} $-algebra and $ n \in \mathbb{N} $. Setting $ \mathcal{D} \stackrel{\text{df}}{=} A^{n} \setminus \{ (0_{A},\ldots,0_{A}) \} $, we can define a function $ f: \...
Transcendental's user avatar
1 vote
0 answers
58 views

Bounds on $\sum_{j=1}^m\frac{\pi^j}{\Gamma(j)(x^2+(j+1/4)^2)}$

During our search of real rooted entire function approximations to Riemann $\Xi$ function, we need to calculate the upper and lower bounds of $$f_m(x):=\sum_{j=1}^m b_j(x):=\sum_{j=1}^m\frac{\pi^j}{\...
mike's user avatar
  • 603
1 vote
0 answers
143 views

Bounding the Schur's complement of similiar matrices

Assume the following: • $L\leq K$ . • $\Gamma\in M_{K,L}$ is a $L$ rank ${ 0,1} $ matrix, without identical rows or the zeros row. • $N\in M_{K,K}$ is a diagonal matrix, whose diagonal is a ...
ifog's user avatar
  • 295
1 vote
0 answers
108 views

Bounding a q-expansion on a bounded open subset of the complex upper-half plane

Let $f:\mathbf{H}\to \mathbf{C}$ be a holomorphic function on the complex upper-half plane and let $q:\tau\mapsto \exp(\pi i \tau)$ be the nome on $\mathbf{H}$. Suppose that there are integers $a_j$ ...
Alfonso's user avatar
  • 11
0 votes
0 answers
43 views

Lower bound for the fractional Sobolev norm of the Hermites function

For $r \in (0, 2)$, I am interested in a lower bound for the quantity : $$I_r(n) := \int_{\mathbb{R}} |f_n(x)|^2 |x|^{r} dx$$ where $f_n(x) = (-1)^n (\sqrt{\pi} n! 2^n)^{-1/2} e^{x^2/2} \dfrac{d^n}{dx^...
jvc's user avatar
  • 183
0 votes
0 answers
46 views

Classifier-specific lower bounds on the misclassification rate in binary classification

Consider a binary classification problem for $(X,Y)$, and let $\hat{f}$ be a proposed classifier. We wish to bound the misclassification rate $P(\hat{f}(X)\ne Y)$. There are many known lower bounds on ...
tim523's user avatar
  • 13
0 votes
0 answers
228 views

Is it possible to bound Mertens function $M(n)$ from an inclusion-exclusion formulation?

In this post I proposed a formulation of Mertens function $M(n)$ using the inclusion-exclusion principle, as follows: $$M(n)=-\pi\left(n\right)+\left(\sum_{p_{i}<\sqrt{n}}\pi\left(\lfloor\frac{n}{...
Juan Moreno's user avatar
0 votes
0 answers
167 views

How many elements have a "small" order in a finite field?

I'm hoping that this is an easy question for someone. How many elements can we expect to have multiplicative order at most $n^{1/c}$ in one of the finite fields $\mathbb{F}_p$ with $p$ prime with $n \...
Matt Groff's user avatar
0 votes
0 answers
240 views

Lower-bound smallest eigenvalue of covariance matrix of $y = f(Ax)$, for $x$ uniform on unit-sphere

Let $A=(a_1,\ldots,a_)$ be a fixed $k \times d$ matrix (with $d$ large), and $x$ be a random vector uniformly distributed on the unit-sphere in $\mathbb R^d$. Let $f:\mathbb R \to \mathbb R$ be a ...
dohmatob's user avatar
  • 6,726
0 votes
0 answers
329 views

Lower-bound on expected value of norm of transformation of random vector with iid Rademacher coordinates

Let $n$ be a large positive integer. Let $A$ be a positive-definite matrix such with eigenvalues $\lambda_1 \ge \lambda_2 \ge \ldots \ge \lambda_n$ such that $\lambda_n = o(1) \to 0$ and $\lambda_i=\...
dohmatob's user avatar
  • 6,726
0 votes
0 answers
125 views

Spread of a disease on a modular chessboard (torus) - lower bound

I learned about the following result from one of Peter Winkler's books: It is impossible to infect the entire $n\times n$ chessboard (usual chessboard) starting from fewer than $n$ infected cells. The ...
Steve's user avatar
  • 1
0 votes
0 answers
236 views

Lower bound of exponential sum

This question is a close cousin of the following: Lower bound on exponential sums Let $\phi:[0,1]\to \mathbb R$ be a smooth function with $\frac 1 {10}<\phi'< 10, \frac 1 {10}<\phi''< 10$. ...
Thomas Yang's user avatar
0 votes
0 answers
91 views

Upper bounding the sum with hypergeometric and binomial probabilities

Could you please help me upper bound this tricky expression: $$P(A)=\sum_{i=0}^n{\left( 1 - \dfrac{\binom kq \binom {n-k}{i-q}}{\binom {n}{i}} \right)}^I \binom ni p^i {(1-p)}^{n-i}$$. So far I only ...
abs135's user avatar
  • 1
0 votes
0 answers
225 views

Bounding the absolute value of a complex integral

I'm working on some problems involving Fourier transforms and convolution problems and there is one problem I cannot solve. In my situation we have a complex valued function $f(ix)$, with $x\in\mathbb{...
seaver's user avatar
  • 129
0 votes
0 answers
64 views

if $\max_{z \in K} |\zeta(z+it)-f(z)|<\epsilon.$ then is this $\lim_{t\to \infty} \inf \frac {|\zeta'(z+it)|}{|f'(z)|} $ a finit limit?

Universality theorem of Riemann zeta function states that :Let $K$ be a compact subset with connected complement lying in the strip $\{1/2 < \operatorname{Re}(z)<1\}$, and let $f : K \rightarrow ...
zeraoulia rafik's user avatar
0 votes
0 answers
228 views

What do square roots as minimums have to do with Harmonic numbers?

In an earlier question where I conjectured (and GH from MO confirmed) that the von Mangoldt function is the limit at s=1 of a certain Dirichlet series: $$\Lambda(m)=\lim_{s\to 1+}\zeta(s)\sum_{d\mid ...
Mats Granvik's user avatar
  • 1,133
0 votes
0 answers
99 views

Lower bound for the probability that $X=\omega\left(\mathbb E[X]\right)$ for $X\sim Bin(n,p)$

Let $X\sim Bin(n,p)$ be a binomial variable and let $\delta\in (0,1)$. I'm looking for a lower bound of the form $\Pr[X > f(\delta)] \ge \delta$. Specifically, if $\delta,p=o(1)$ are not ...
R B's user avatar
  • 608
0 votes
0 answers
130 views

Estimating when does a certain binomial sum exceed an upper bound

Given a fixed integer $n > 0$ and $0 \le m \le n$ let us define the numbers $$f_{n,m} = \sum_{i=\lfloor m/2 \rfloor}^m {n-2i \choose n - m -i}{i+1 \choose m - i +1}.$$ For example $f_{n,0} = 1,f_{...
Jernej's user avatar
  • 3,433